Đến nội dung

dark templar

dark templar

Đăng ký: 06-09-2010
Offline Đăng nhập: Riêng tư
****-

#625507 $\sum \frac{a(b+c)}{a^{2}+(b+c)^...

Gửi bởi dark templar trong 06-04-2016 - 21:38

Cho a, b, c>0. Tìm GTLN của: $\sum \frac{a(b+c)}{a^{2}+(b+c)^{2}}$

Đặt $P=\sum \frac{a(b+c)}{a^2+(b+c)^2}$

 

Biến đổi $\frac{1}{2}-\frac{a(b+c)}{a^{2}+\left ( b+c \right )^{2}}=\frac{(b+c-a)^{2}}{2(a^{2}+(b+c)^{2})}$

 

Do đó $3-2P=\sum \frac{(b+c-a)^{2}}{a^{2}+(b+c)^{2}}$.Ta sẽ chứng minh $\sum \frac{(b+c-a)^{2}}{a^{2}+(b+c)^{2}}\geqslant \frac{3}{5}$

 

Đặt $x=\frac{b+c}{a};y=\frac{c+a}{b};z=\frac{a+b}{c}$ thì $x,y,z>0$.BĐT cần chứng minh tương đương với $\sum \frac{(x-1)^{2}}{x^{2}+1}\geqslant \frac{3}{5}$

 

Theo C-S:

$$\sum \frac{(x-1)^{2}}{x^{2}+1}\geqslant \frac{(x+y+z-3)^{2}}{x^{2}+y^{2}+z^{2}+3}$$

 

Ta chỉ cần chứng minh:

$$\frac{(x+y+z-3)^{2}}{x^{2}+y^{2}+z^{2}+3}\geqslant \frac{3}{5}\Leftrightarrow \left ( \sum x \right )^{2}-15\sum x+3\sum xy+18\geqslant 0$$

 

Mặt khác,theo BĐT Schur:

$$\frac{(a+b)(b+c)}{ac}+\frac{(b+c)(c+a)}{ab}+\frac{(c+a)(a+b)}{bc}\geqslant 2\left ( \frac{a+b}{c}+\frac{b+c}{a}+\frac{c+a}{b} \right ) \Leftrightarrow \sum xy \geqslant 2\sum x$$

 

Do đó:

$$\left ( \sum x \right )^{2}-15\sum x+3\sum xy+18\geqslant \left ( \sum x \right )^{2}-9\sum x+18=\left ( \sum x -6\right )\left ( \sum x -3 \right )\geqslant 0$$ 

 

do $\sum x \geqslant 6$

 

Vậy $P \leqslant \frac{6}{5}$ .Đẳng thức xảy ra khi $x=y=z$ hay $a=b=c$.




#625377 Chứng minh rằng: $2(x+y+z)-xyz\leq 10$

Gửi bởi dark templar trong 06-04-2016 - 17:11

1. Cho x,y,z là các số thực thỏa mãn $x^2+y^2+z^2=9, xyz\leq 0$ $\leq 0$.

Chứng minh rằng: $2(x+y+z)-xyz\leq 10$

2. Cho số thực $k\in (0,2015)$. Xét các số thực không âm x,y,x thỏa mãn x+y+z=1. Tùy theo k tìm GTLN của biểu thức: E=xy+yz+zx-kxyz

Bài 1 này là VMO 2002,bạn có thể sử dụng Google để tìm lời giải.

 

Bài 2 này nếu đã biết tới BĐT Schur thì sẽ rất dễ dàng để xử lý.

Xét $q=ab+bc+ca,r=abc$ thì $0<q \leqslant \frac{1}{3}$ và $0 \leqslant r \leqslant \frac{1}{27}$

 

Theo Schur thì $r\geqslant \min \left \{ 0;\frac{4q-1}{9} \right \}$ nên ta sẽ xét 2 trường hợp:

 

$0<q \leqslant \frac{1}{4}:$ Khi này $r \geqslant 0$ nên $E \leqslant q \leqslant \frac{1}{4}$

 

$\frac{1}{4} \leqslant q \leqslant \frac{1}{3}:$ Khi này $r \geqslant \frac{4q-1}{9}$.Do đó $E \leqslant q-\frac{k(4q-1)}{9}=\frac{(9-4k)q}{9}+\frac{k}{9}$

 

Đến đây ta lại xét 2 trường hợp nhỏ là $0<k \leqslant \frac{9}{4}$ và $k>\frac{9}{4}$.

 

Với $0<k \leqslant \frac{9}{4}$ thì $E\leqslant \frac{\frac{9-4k}{3}+k}{9}=\frac{9-k}{27}$

 

Với $k>\frac{9}{4}$ thì $E\leqslant \frac{\frac{9-4k}{4}+k}{9}=\frac{1}{4}$

 

Như vậy ta có thể kết luận như sau:

 

Với $0<k \leqslant \frac{9}{4}$ thì $\max E=\frac{9-k}{27}$

Với $2015>k>\frac{9}{4}$ thì $\max E=\frac{1}{4}$

 

Nhận xét ngoài lề:

Ngoài ra ta còn có thể tìm GTNN của $E$ phụ thuộc vào $k$.Ở đây là link cho 1 topic tập hợp kết quả của các dạng toán này:

http://www.artofprob...about_abbccaabc




#625321 Tìm GTNN của P= $\frac{x}{y+z}+\frac{...

Gửi bởi dark templar trong 06-04-2016 - 11:04

Cho $x,y,z$ là các số thực dương thỏa mãn $\frac{1}{x^2}+\frac{1}{y^2}=\frac{1}{2z^2}$. Tìm GTNN của P= $\frac{x}{y+z}+\frac{y}{z+x}+\frac{z}{\sqrt{x^2+y^2+z^2}}$

Điều kiện tương đương với $z=\frac{xy}{\sqrt{2(x^{2}+y^{2})}} \leqslant \frac{xy}{x+y}$.Theo C-S thì:

$$\frac{x}{y+z}+\frac{y}{z+x}\geqslant \frac{(x+y)^{2}}{2xy+z(x+y)}$$

 

Do đó:

$P\geqslant \frac{(x+y)^{2}}{3xy}+\frac{1}{\sqrt{\frac{x^{2}+y^{2}}{z^{2}}+1}}=\frac{x^{2}+y^{2}}{3xy}+\frac{2}{3}+\frac{xy}{\sqrt{2(x^{2}+y^{2})^{2}+x^{2}y^{2}}}$

$\geqslant \frac{x^{2}+y^{2}}{3xy}+\frac{2}{3}+\frac{xy}{\sqrt{2(x^{2}+y^{2})^{2}+\frac{(x^{2}+y^{2})^{2}}{4}}}=\frac{x^{2}+y^{2}}{3xy}+\frac{2}{3}+\frac{2xy}{3(x^{2}+y^{2})}$

 

Đặt $t=\frac{x^2+y^2}{xy}$ thì $t \geqslant 2$.Khi đó $P \geqslant \frac{2}{3}+\frac{1}{3}f(t)$ với $f(t)=t+\frac{2}{t}$

 

Đạo hàm $f'(t)=1-\frac{2}{t^2}>0,\forall t \geqslant 2$ nên $f(t) \geqslant f(2)=3$,suy ra $P \geqslant \frac{5}{3}$

 

$P_{\min}=\frac{5}{3}$ khi và chỉ khi $x=y=2z$




#625314 CMR $xyz(x+y+z)\geq 2(xy+yz+zx)$.

Gửi bởi dark templar trong 06-04-2016 - 10:19

Từ ĐK đề bài cho,tồn tại các số $a,b,c>0$ sao cho $x=\sqrt{\frac{a+b}{c}};y=\sqrt{\frac{b+c}{a}};z=\sqrt{\frac{c+a}{b}}$.

 

Do đó BĐT cần chứng minh trở thành:

$$\sqrt{\frac{a+b}{c}}+\sqrt{\frac{b+c}{a}}+\sqrt{\frac{c+a}{b}}\geqslant 2\left ( \sqrt{\frac{a}{b+c}}+\sqrt{\frac{b}{c+a}}+\sqrt{\frac{c}{a+b}} \right )$$

 

Ta có:

$$\sum \sqrt{\frac{b+c}{a}}\geqslant \sum \frac{\sqrt{b}+\sqrt{c}}{\sqrt{2a}}=\frac{1}{\sqrt{2}}\sum \left ( \frac{\sqrt{b}}{\sqrt{a}}+\frac{\sqrt{b}}{\sqrt{c}} \right )\geqslant \sqrt{2}\sum \sqrt{\frac{b}{\sqrt{ac}}}\geqslant 2\sum \sqrt{\frac{b}{a+c}}$$

 

Ta có đpcm.




#625293 Cho tam giác $ABC$ thỏa mãn $cos^{2}B+ cos^{2...

Gửi bởi dark templar trong 06-04-2016 - 01:43

 

Câu 4: 

           a) Cho tam giác $ABC$ thỏa mãn $cos^{2}B+ cos^{2}C\leq sin^{2}A$

           Tìm giá trị lớn nhất của biểu thức: $F=\sqrt{2}sin^{4}\frac{A}{2}+\sqrt{2}cos^{2}\frac{A}{2}+cos^{2}\frac{B}{2}+cos^{2}\frac{C}{2}$

# Thảo luận về đề thi HSG 11 tỉnh Hà Tĩnh 2015- 2016 tại đây.

 

Theo C-S:

$$\sin^{2}A\geqslant \cos^{2}B+\cos^{2}C\geqslant \frac{\left ( \cos B+\cos C \right )^{2}}{2}\Leftrightarrow \sqrt{2}\sin A\geqslant \left | \cos B+\cos C \right |\geqslant \cos B+\cos C$$

 

Biến đổi:

$$F=\sqrt{2}\sin^{4}\frac{A}{2}+\sqrt{2}\cos^{2}\frac{A}{2}+1+\frac{\cos B+\cos C}{2}\leqslant \sqrt{2}\left ( \sin^{4}\frac{A}{2}+1-\sin^{2}\frac{A}{2}+2\sin\frac{A}{2}\cos\frac{A}{2}\right)+1$$

 

Đặt $t=\sin\frac{A}{2}$ thì $\cos\frac{A}{2}=\sqrt{1-t^2}$ và $0<t<1$.Khi đó $F \leqslant \sqrt{2}f(t)+1$ với $f(t)=t^4+1-t^2+2t\sqrt{1-t^2}$

 

Khảo sát hàm $f(t)$ trên khoảng $(0,1$ với $f'(t)=\frac{2\left ( 2t^{2}-1 \right )\left ( t\sqrt{1-t^{2}}-1 \right )}{\sqrt{1-t^{2}}}$

 

Để ý rằng $t\sqrt{1-t^2}-1<0,\forall t \in (0,1)$ nên $f'(t)=0$ có nghiệm là $t=\frac{1}{\sqrt{2}}$ và $f'(t)$ trái dấu với $2t^2-1$.

 

Do đó $\max_{0<t<1} f(t)=f\left(\frac{1}{\sqrt{2}} \right)=\frac{7}{4}$.

 

Vậy ta có $F \leqslant \sqrt{2}f(t)+1 \leqslant 1+\frac{7\sqrt{2}}{4}$.$F_{\max}=1+\frac{7\sqrt{2}}{4}$ khi $B=C=\frac{\pi}{4};A=\frac{\pi}{2}$




#625289 Tính $limS_{n}$.

Gửi bởi dark templar trong 06-04-2016 - 00:46

Ta có thể viết lại công thức truy hồi dưới dạng :

$$u_{n+1}=u_{n}+\frac{2u_n}{2n-1}=\frac{2n+1}{2n-1}u_n=\frac{2n+1}{2n-1}.\frac{2n-1}{2n-3}u_{n-1}=....=\frac{2n+1}{2.1-1}u_1=2(2n+1)$$

 

Vậy :

$\lim \frac{1}{n^{3}}\sum_{k=1}^{n}u_{k}^{2}=\lim \frac{4}{n^{3}}\left ( \sum_{k=1}^{n}\left ( 2k-1 \right )^{2} \right )=\lim \frac{4}{n^{3}}\left ( 4\sum_{k=1}^{n}k^{2}+n-4\sum_{k=1}^{n}k \right )$

$=\lim \frac{4}{n^{3}}\left ( \frac{2n(n+1)(2n+1)}{3}+n-2n(n+1) \right )=\frac{16}{3}$

 




#625287 Tìm $Min \sum \frac{x}{y^2+z^2}$

Gửi bởi dark templar trong 06-04-2016 - 00:15

Tìm MIN của 

$\sum \frac{x}{y^{2}+z^{2}}$

với x;y;z>0 và $x^2+y^2+z^2=1$

Chứng minh thông qua BĐT phụ $\frac{x}{1-x^{2}}\geqslant \frac{3\sqrt{3}x^{2}}{2},\forall x>0$ và để ý $\sum \frac{x}{y^{2}+z^{2}}=\sum_{x,y,z}\frac{x}{1-x^{2}}$




#625078 Inequalities From 2016 Mathematical Olympiads

Gửi bởi dark templar trong 05-04-2016 - 17:20

 

Bài 12 (China Junior High School Mathematics League). Với $x,\,y,\,z$ là ba số thực dương thỏa mãn $xy+yz+zx\neq 1$ và $\frac{(x^2-1)(y^2-1)}{xy} +\frac{(y^2-1)(z^2-1)}{yz} +\frac{(z^2-1)(x^2-1)}{zx} =4.$ Prove that $$9(x+y)(y+z)(z+x)\geqslant 8xyz(xy+yz+zx).$$

Biến đổi điều kiện đề bài cho ,ta sẽ có:

$\sum x\left ( y^{2}-1 \right )\left ( z^{2}-1 \right )=4xyz\Leftrightarrow xyz\sum xy+\sum x-\sum xy(x+y)=4xyz$

$\Leftrightarrow xyz\sum xy+\left ( \sum x-xyz \right )=\left ( \sum x \right )\left ( \sum xy \right )\Leftrightarrow \left ( \sum xy -1 \right )\left ( \sum x -xyz\right )=0$

$\Leftrightarrow x+y+z=xyz$

 

Đổi biến $\left ( a,b,c \right )\rightarrow \left ( \frac{1}{x},\frac{1}{y},\frac{1}{z} \right )$ thì ta có $a,b,c>0$ và $ab+bc+ca=1$.

 

BĐT cần chứng minh trở thành:

$$9(a+b)(b+c)(c+a)\geqslant 8(a+b+c)$$

 

Viết dưới dạng thuần nhất :

$$\left ( a+b \right )(b+c)(c+a)\geqslant \frac{8}{9}(a+b+c)(ab+bc+ca)$$

 

BĐT này chỉ là hệ quả của $(a+b)(b+c)(c+a)=(a+b+c)(ab+bc+ca)-abc$ và $abc\leqslant \frac{(a+b+c)(ab+bc+ca)}{9}$.Ta có đpcm.




#624990 Tìm min,max: $\sum \frac{x+y}{1+z}$

Gửi bởi dark templar trong 05-04-2016 - 10:15

Cho x, y, z trong khoảng từ $\frac{1}{2}$ đến 1. Tìm GTNN và GTLN của:

$\frac{x+y}{1+z}+\frac{y+z}{1+x}+\frac{z+x}{1+y}$

Xem ở http://www.artofprob...t_round_problem




#624987 $\sum \frac{a^2+16bc}{b^2+c^2}\geq 10...

Gửi bởi dark templar trong 05-04-2016 - 10:10

Cho a,b,c là các số không âm.Chứng minh rằng:

$\sum \frac{a^2+16bc}{b^2+c^2}\geq 10$

Do bài này nằm trong box THCS nên mình sẽ đưa bạn lời giải phù hợp ở http://artofproblems...1199960p5898149 (xem bài viết #7 )




#624984 Inequalities From 2016 Mathematical Olympiads

Gửi bởi dark templar trong 05-04-2016 - 08:26

 Bài 2. Ta chú ý đến một phân tích quen thuộc là $(x+y)^2=(x-y)^2+4xy$

 Bây giờ đặt $\sum _{j=1}^na_jb_j=S$ và $a_{i}b_{i}=S_i$ thì ta cần chứng minh $\sum_{i=1}^n \dfrac{(a_{i+1}+b_{i+1})^2}{n(a_i-b_i)^2+4(n-1)S} \geq \dfrac{1}{n-1}$

 Hay $\sum_{i=1}^n \dfrac{(a_{i+1}-b_{i+1})^2+4S_{i+1}}{n(a_i-b_i)^2+4(n-1)S} \geq \dfrac{1}{n-1}$

 Không mất tính tổng quát giả sử $(a_1-b_1)^2\leq \min \left \{(a_2-b_2)^2,(a_3-b_3)^2,...,(a_n-b_n)^2\right \}$ thì ta có

 $\sum_{i=1}^n \dfrac{(a_{i+1}-b_{i+1})^2+4S_{i+1}}{n(a_i-b_i)^2+4(n-1)S} \geq \dfrac{\sum \limits_{i=1}^n (a_{i}-b_{i})^2+4\sum \limits_{k=1}^nS_{k}}{n(a_1-b_1)^2+4(n-1)S }(1)=\dfrac{\sum \limits_{i=1}^n (a_{i}-b_{i})^2+4S}{n(a_1-b_1)^2+4(n-1)S}$

 Nên ta chỉ cần chứng minh $\dfrac{\sum \limits_{i=1}^n (a_{i}-b_{i})^2+4S}{n(a_1-b_1)^2+4(n-1)S}\geq \dfrac{1}{n-1}\Leftrightarrow \sum_{i=1}^n (a_{i}-b_{i})^2\geq \dfrac{n}{n-1}(a_1-b_1)^2\Leftrightarrow \sum_{i=2}^n (a_{i}-b_{i})^2\geq \dfrac{1}{n-1}\geq (a_1-b_1)^2$

 Áp dụng bất đẳng thức Cauchy-Schwarz ta có $\sum_{i=2}^n (a_{i}-b_{i})^2\geq \dfrac{1}{n-1}\left (\sum_{j=2}^n a_j-\sum_{k=2}^n b_k \right )^2 (2)$

 Mặt khác $\sum_{i=1}^n (a_i-b_i)=0$ nên $\left (\sum_{j=2}^n a_j-\sum_{k=2}^n b_k \right )^2=(a_1-b_1)^2$

 Từ đó ta có điều cần chứng minh ~.~ Mỏi tay

Chỗ khúc $(1)$ thì tử số của mẫu chỉ là $\sum_{k=1}^{n-1}\left ( a_{k}-b_{k} \right )^{2}+\left ( a_{n+1}-b_{n+1} \right )^{2}+\sum_{k=2}^{n}S_{k}+b_{n+1}a_{n+1}$.Đó là lý do mình hỏi có cái quy ước nào cho $a_{n+1}$ và $b_{n+1}$ hay không  :mellow:

 

Còn khúc $(2)$ thì nên viết rõ là $\sum_{i=2}^{n}\left ( a_{i}-b_{i} \right )^{2}\geqslant \frac{1}{n-1}\left ( \sum_{i=2}^{n}\left | a_{i}-b_{i} \right | \right )\geqslant \frac{1}{n-1}\left ( \sum_{k=2}^{n}a_{k}-\sum_{j=2}^{n}b_{j} \right )$




#624961 $\Delta ABC.CM: (b^{2} - c^{2})cotA + (c^{...

Gửi bởi dark templar trong 04-04-2016 - 23:26

$Cho \Delta ABC với góc A,B,C. Các cạnh AB = c, AC = b, BC = a.
\\CMR: (b^{2} - c^{2})cotA + (c^{2} - a^{2})cotB + (a^{2} - b^{2})cotC = 0 (1)$

Biến đổi thuần Đại Số:

 

$$\left ( b^{2}-c^{2} \right )\cot A=\frac{\left ( b^{2}-c^{2} \right )2bc\cos A}{2bc\sin A}=\frac{\left ( b^{2}-c^{2} \right )\left ( b^{2}+c^{2}-a^{2} \right )}{4S}$$

 

Ta cần chứng minh:

$$\left ( b^{2}-c^{2} \right )\left ( b^{2}+c^{2}-a^{2} \right )+\left ( c^{2}-a^{2} \right )\left ( c^{2}+a^{2}-b^{2} \right )+\left ( a^{2}-b^{2} \right )\left ( a^{2}+b^{2}-c^{2} \right )=0$$

 

Hay:

$$\left ( b^{4}-c^{4} \right )+\left ( c^{4}-a^{4} \right )+\left ( a^{4} -b^{4}\right )+\left ( a^{2}c^{2}-a^{2}b^{2} \right )+\left ( a^{2}b^{2}-b^{2}c^{2} \right )+\left ( b^{2}c^{2}-a^{2}c^{2} \right )=0$$

 

Đẳng thức trên luôn đúng nên ta có đpcm.

 

Nếu ta thay hàm $\cot$ bằng hàm $\tan$ thì đẳng thức trên vẫn đúng :)




#624948 Chứng minh rằng: $a^3 +b^3+c^3 \geq \frac{3}{8...

Gửi bởi dark templar trong 04-04-2016 - 22:45

Cho 3 số dương a,b,c thỏa mãn $a^2 +b^2 +c^2 +2abc = 1$

Chứng minh rằng: $a^3 +b^3+c^3 \geq \frac{3}{8}$

Sử dụng AM-GM để hạ bậc $a^{3}+a^{3}+\frac{1}{8}\geqslant \frac{3}{2}a^{2}$,ta chỉ cần chứng minh $a^{2}+b^{2}+c^{2}\geqslant \frac{3}{4}$

 

Từ điều kiện đề bài cho ,ta có:

$$a^{2}+b^{2}+c^{2}=1-2abc=1-2\sqrt{a^{2}b^{2}c^{2}}\geqslant 1-2\sqrt{\left ( \frac{a^{2}+b^{2}+c^{2}}{3} \right )^{3}}(*)$$

 

Đặt $t=\sqrt{\frac{a^{2}+b^{2}+c^{2}}{3}}\left ( t>0 \right )$ thì:

$$(*)\Leftrightarrow 3t^{2}\geqslant 1-2t^{3}\Leftrightarrow \left ( t+1 \right )^{2}\left ( 2t-1 \right )\geqslant 0\Leftrightarrow t\geqslant \frac{1}{2}$$

 

Hay $a^{2}+b^{2}+c^{2}\geqslant \frac{3}{4}$.Ta có đpcm.




#624929 $\sum \frac{a\sqrt{bc}.IA}{b.IB^...

Gửi bởi dark templar trong 04-04-2016 - 22:13

Cho tam giác $ABC$ có $I$ là tâm đường tròn nội tiếp ,các cạnh $BC=a,AC=b,AB=c$ 
Chứng minh các bất đẳng thức sau : 
ii) $\sum \frac{a\sqrt{bc}.IA}{b.IB^2+c.IC^2} \ge \frac{3\sqrt{3}}{2}$

Ta có hệ thức sau :

$$aIA^2+bIB^2+cIC^2=abc$$

 

Hệ thức này có khá nhiều cách chứng minh nên nhường lại cho chủ topic.

 

Từ đó ta có:

$$\frac{a\sqrt{bc}IA}{bIB^{2}+cIC^{2}}=\frac{a\sqrt{bc}IA}{abc-aIA^{2}}=\frac{\sqrt{bc}IA}{bc-IA^{2}}$$

 

Mặt khác:

$$IA=\frac{p-a}{\cos\frac{A}{2}}=\frac{p-a}{\sqrt{\frac{1+\cos A}{2}}}=\frac{p-a}{\sqrt{\frac{p(p-a)}{bc}}}=\sqrt{\frac{bc(p-a)}{p}}$$

 

với ký hiệu $p$ là nửa chu vi

 

Do đó:

$$\frac{a\sqrt{bc}IA}{bIB^{2}+cIC^{2}}=\frac{bc\sqrt{\frac{p-a}{p}}}{bc\left ( 1-\frac{p-a}{p} \right )}=\frac{\sqrt{p(p-a)}}{a}$$

 

Đặt $x=\sqrt{p-a};y=\sqrt{p-b};z=\sqrt{p-c}$ thì $x,y,z>0$ và $a=y^2+z^2;b=x^2+z^2;c=x^2+y^2$.BĐT sẽ tương đương với:

$$\frac{x}{y^{2}+z^{2}}+\frac{y}{x^{2}+z^{2}}+\frac{z}{x^{2}+y^{2}}\geqslant \frac{3\sqrt{3}}{2\sqrt{x^{2}+y^{2}+z^{2}}} (*)$$

 

Mình có 2 cách để chứng minh $(*)$.

 

Cách 1: Sử dụng tính thuần nhất của BĐT,chuẩn hoá cho $x^2+y^2+z^2=3$.Viết lại BĐT dưới dạng sau:

$$\frac{x}{3-x^{2}}+\frac{y}{3-y^{2}}+\frac{z}{3-z^{2}}\geqslant \frac{3}{2}$$

 

Ta có BĐT phụ sau : $\frac{x}{3-x^{2}}\geqslant \frac{x^{2}}{2},\forall x>0 \Leftrightarrow (x-1)^2(x+2) \ge 0$.Xây dựng 2 BĐT tương tự cho $y,z$ rồi cộng chúng lại,ta sẽ có đpcm

---------------------

 

Cách 2 (chứng minh trực tiếp)

Ta cần chứng minh $\left ( \sum \frac{x}{y^{2}+z^{2}} \right )^{2}\geqslant \frac{27}{4(x^{2}+y^{2}+z^{2})}$

 

Theo Holder thì:

$$\left ( \sum \frac{x}{y^{2}+z^{2}} \right )^{2}\left ( \sum x^{4}(y^{2}+z^{2})^{2} \right )\geqslant \left ( x^{2}+y^{2}+z^{2} \right )^{3}$$

 

Như vậy ta chỉ cần chứng minh:

$$2\left ( x^{2}+y^{2}+z^{2} \right )^{4}\geqslant 27\left ( x^{4}y^{4}+y^{4}z^{4}+z^{4}x^{4}+x^{2}y^{2}z^{2}\left ( x^{2}+y^{2}+z^{2} \right ) \right )$$

 

Đổi biến $x^2=a,y^2=b,z^2=c(a,b,c>0)$ cho dễ nhìn thì ta cần chứng minh:

$$2\left ( a+b+c \right )^{4}\geqslant 27\left ( a^{2}b^{2}+b^{2}c^{2}+c^{2}a^{2}+abc(a+b+c) \right )$$

 

Tương đương với:

$$2\sum a^{4}+8\sum ab(a^{2}+b^{2})\geqslant 15\sum a^{2}b^{2}+3abc(a+b+c)$$

 

Theo AM-GM:

$$VT\geqslant 18\sum a^{2}b^{2}\geqslant 15\sum a^{2}b^{2}+3abc(a+b+c)=VP$$

 

Vậy ta có đpcm.

 




#624685 Inequalities From 2016 Mathematical Olympiads

Gửi bởi dark templar trong 03-04-2016 - 22:34

Chào các bạn như tiêu đề đã ghi thì trong topic này chúng ta sẽ cùng thảo luận về các bài toán bất đẳng thức đến từ các cuộc thi học sinh giỏi Toán khắp nơi trên thế giới trong năm 2016 và kết thúc topic này (có thể là sau IMO 2016) chúng ta sẽ có một file pdf “Inequalities From 2016 Mathematical” made in VMF để các thành viên có tài liệu tham khảo, hi vọng các bạn ủng hộ. :)

 

Bài 2 (Korea Winter Program Practice Test). Với số nguyên dương $n \geqslant 2$ và $a_i,\,b_i\;(1 \leqslant i \leqslant n)$ là các số thực dương thỏa mãn $\sum_{i=1}^n a_i = \sum_{i=1}^n b_i.$ Chứng minh rằng 

\[\sum_{i=1}^n \frac{(a_{i+1}+b_{i+1})^2}{n(a_i-b_i)^2+4(n-1)\displaystyle \sum_{j=1}^n a_jb_j} \geqslant \frac{1}{n-1}.\]

 

 

Bạn Huyện cho mình hỏi là bài này có ký hiệu $a_{n+1}=a_1$ và $b_{n+1}=b_1$ không vậy ?